S&Jv8,p27.01: Add missing period to the question
[course.git] / latex / problems / Serway_and_Jewett_8 / problem05.25.tex
1 \begin{problem*}{5.25}
2 A bag of cement whose weight is $F_g$ hangs in equilibrium from three
3 wires shown in Figure~P5.24.  Two of the wires make angles
4 $\theta_1=60.0\dg$ and $\theta_2=40.0\dg$ with the horizontal.
5 Assuming the system is in equilibrium, show that the tension in the
6 left-hand wire is
7 \begin{equation}
8   T_1 = \frac{F_g \cos\theta_2}{\sin(\theta_1+\theta_2)}
9 \end{equation}
10 \begin{center}
11 \begin{asy}
12 import Mechanics;
13
14 real u = 1cm;
15
16 real theta1 = 60;
17 real theta2 = 40;
18 real d = 3u;      // contact point seperation
19 real a = u;       // block side length
20
21 pair c1 = (0,0);       // left contact point
22 pair c2 = c1 + (d,0);  // right contact point
23 pair c3 = extension(c1, c1+dir(-theta1),
24                     c2, c2+dir(180+theta2));
25
26 Surface s = Surface(c2+(0.2d,0), c1-(0.2d,0));
27 Angle a1 = Angle(c2, c1, c3, "$\theta_1$");
28 Angle a2 = Angle(c1, c2, c3, "$\theta_2$");
29 Block cement = Block(c3-(0, a), width=a, height=a, "$F_g$");
30
31 a1.draw();
32 a2.draw();
33 draw(c1 -- c3 -- c2);
34 draw(c3 -- cement.center);
35 s.draw();
36 cement.draw();
37
38 label("$T_1$", (c1+c3)/2, SW);
39 label("$T_2$", (c2+c3)/2, SE);
40 label("$T_3$", c3-(0,a/4), E);
41 \end{asy}
42 \end{center}
43 \end{problem*}
44
45 \begin{solution}
46 Balancing forces on the cement bag, $T_3=F_g$.
47
48 Balancing forces on the wire joint is a bit mor complicated and
49 deserves a free body diagram.
50 \begin{center}
51 \begin{asy}
52 import Mechanics;
53
54 real u = 1cm;
55
56 real theta1 = 60;
57 real theta2 = 40;
58 real T3 = 2u;
59 real T1 = T3 * Cos(theta2)/Sin(theta1+theta2);
60 real T2 = T3 * Cos(theta1)/Sin(theta1+theta2);
61
62 Angle a1 = Angle(dir(180-theta1), (0,0), (-1,0), "$\theta_1$");
63 Angle a2 = Angle(dir(theta2), (0,0), (1,0), "$\theta_2$");
64 Angle A1 = Angle(dir(180-theta1), (0,0), dir(180+theta2), radius=10mm,
65     "$\theta_1+\theta_2$");
66 Angle A3 = Angle((0,-1), (0,0), dir(-90+theta2), "$\theta_2$");
67
68 Vector F1 = Force((0,0), mag=T1, dir=180-theta1, "$\vect{F}_1$");
69 Vector F2 = Force((0,0), mag=T2, dir=theta2,
70     L=Label("$\vect{F}_2$", position=EndPoint, align=N));
71 Vector F3 = Force((0,0), mag=T3, dir=-90, "$\vect{F}_3$");
72
73 a1.draw();
74 a2.draw();
75 A1.draw();
76 A3.draw();
77 draw((-7mm,0)--(7mm,0));
78 draw((12mm*dir(-90+theta2))--(0,0)--(12mm*dir(180+theta2)));
79 F1.draw();
80 F2.draw();
81 F3.draw();
82 dot((0,0));
83
84 draw_ijhat((1.5u,-.5u), theta2);
85 \end{asy}
86 \end{center}
87 Where we've chosen a coordinate system such that $\vect{F}_2$ has no
88 $y$ component.
89
90 Balancing forces in the $y$ direction,
91 \begin{align}
92   0 &= T_1 \sin(\theta_1+\theta_2) - T_3\cos(\theta_2) \\
93   T_1 &= T_3\frac{\cos(\theta_2)}{\sin(\theta_1+\theta_2)}
94     = F_g\frac{\cos(\theta_2)}{\sin(\theta_1+\theta_2)} \;,
95 \end{align}
96 which is what we set out to show.
97 \end{solution}